Kirchoff's Laws, nodes and current

In summary: Are you saying that what I posed (the last one) is not the right answer?If that's the case, let me know what the right one is.It is VERY straightforward... I have no idea why your TA is saying that. You basically have to remember two things.1) the voltage around a loop is equal to zero.2) current in equals current out.So for your second loop you have:-E2 + I2R2 + I3R3 = 0where I2 is going in the opposite direction of the resistor, so you have a negative term. You are going to have to use the first loop equation to solve for I3 in terms of I1. So you will have I
  • #1
Alt+F4
306
0
http://img157.imageshack.us/img157/2147/showmegf4.gif








a) What is the current I1 which flows through R1?

I1 =

(b) What is the current I3 that flows through R3?









3 Equations

I1-I3 = 0

I1R14 + I1R2 - E1 =0

I3R2 + I3R3 +E2 = 0



is that even right?
 
Last edited by a moderator:
Physics news on Phys.org
  • #2
Alt+F4 said:
http://img157.imageshack.us/img157/2147/showmegf4.gif








a) What is the current I1 which flows through R1?

I1 =

(b) What is the current I3 that flows through R3?









3 Equations

I1-I3 = 0

I1R14 + I1R2 - E1 =0

I3R2 + I3R3 +E2 = 0



is that even right?

Not quite.
Label the node in between R1, R3, and R2. What can you say about this node in terms of current?

From kirchhoffs law you know that
[tex] \sum i_{in} = \sum i_{out} [/tex]

The trouble you are having is that there is another current. Can you see why?

Looking at that node we talked about.

[tex] I_1 = I_2 + I_3 [/tex]
where [itex] I_2 [/itex] is defined as leaving the node.
 
Last edited by a moderator:
  • #3
FrogPad said:
Not quite.
Label the node in between R1, R3, and R2. What can you say about this node in terms of current?

From kirchhoffs law you know that
[tex] \sum i_{in} = \sum i_{out} [/tex]

The trouble you are having is that there is another current. Can you see why?

Looking at that node we talked about.

[tex] I_1 = I_2 + I_3 [/tex]
where [itex] I_2 [/itex] is defined as leaving the node.
so is this how?

I1 = I2 + I3
I1R14 + I2R2 - E1 =0
I2R2 + I3R3 +E2 = 0
 
Last edited:
  • #4
For the last equation, you want to make sure you keep the signs right. You can indicate a drop of voltage as a result of resistors as NEGATIVE, while an increase in PD as a result of a cell as POSITIVE or vice versa, just keep it CONSISTENT
 
  • #5
silver-rose said:
For the last equation, you want to make sure you keep the signs right. You can indicate a drop of voltage as a result of resistors as NEGATIVE, while an increase in PD as a result of a cell as POSITIVE or vice versa, just keep it CONSISTENT
So

I1 = I2 + I3
I1R14 + I2R2 - E1 =0
-I2R2 - I3R3 +E2 = 0


i tried that and it is not the right answer
 
  • #6
Ok so I am guessin it is something like that
I1 = I2 + I3
-E1 + I1R1 + I2R2 + E2 + I1R4 = 0

wat bout the 3rd equation? Thanks
 
  • #7
Alt+F4 said:
wat bout the 3rd equation? Thanks

try writing KVL for the other loop
 
  • #8
Actually
I1 = I2 + I3
-E1 + I1R1 + I2R2 + E2 + I1R4 = 0
E2+R2I2+R3I3
 
  • #9
Alt+F4 said:
Actually
I1 = I2 + I3
-E1 + I1R1 + I2R2 + E2 + I1R4 = 0
E2+R2I2+R3I3


HW is due in 2 hrs and i tried everything any help is appreciated :redface:
 
  • #10
Alt+F4 said:
HW is due in 2 hrs and i tried everything any help is appreciated :redface:

The two loop equations are:

[tex] -\xi_1 +I_1(R_1) +I_3(R_2) +\xi_2 +I_1(R_1) = 0 [/tex]
[tex] -\xi_2 -I_3(R_2) +I_2(R_3) = 0 [/tex]

Note: That is two equations with THREE unknowns. So you need to come up with another equation to solve.
 
  • #11
Thanks Alot
 
  • #12
Try to do this when you are coming up with the equations.

1) First draw the currents (you can choose whatever direction you want).
- Remember that current does not change when passing elements in a series. - It only changes when it reaches elements in parallel.

2) Traverse a branch (ie make a loop).
- When you get to an element (such as a resistor) if the current direction is pointing into the element then write that voltage term (ie V=IR, where I = current going into the element, and R = the resistance of the element) as POSITIVE.
- If you are going around the loop and you reach an element but the current is pointing in the opposite direction of the element, then write that term as NEGATIVE.
- Set all of these terms equal to 0. Take note that the voltage supplied (such as from a voltage source) is equal to the voltage drops (such as a resistor takes away a certain amount of voltage).

You really need to do a few examples for all of this to sink in.
 
  • #13
FrogPad said:
The two loop equations are:

[tex] -\xi_1 +I_1(R_1) +I_3(R_2) +\xi_2 +I_1(R_1) = 0 [/tex]
[tex] -\xi_2 -I_3(R_2) +I_2(R_3) = 0 [/tex]

Note: That is two equations with THREE unknowns. So you need to come up with another equation to solve.
ok sorry but this equation for some reason did not work, i know one equation should be


I1 = I2 + I3
-E1 + I1R14 + I2R2 + E2 = 0



I just still can't get the third equation for the left loop

i was thinking it was

-E2 + R2I2 + R2I3 + R3I3

any help, thanks
 
  • #14
Alt+F4 said:
ok sorry but this equation for some reason did not work, i know one equation should be


I1 = I2 + I3
-E1 + I1R14 + I2R2 + E2 = 0



I just still can't get the third equation for the left loop

i was thinking it was

-E2 + R2I2 + R2I3 + R3I3

any help, thanks

Woops sorry buddy. I interchanged I2 and I3.

The correct equations for this network are:

-E1 +I1R1 +I2R2 +E2 +I1R4 = 0
-E2 -I2R2 +I3R3 = 0
I1 = I2 +I3
 
  • #15
FrogPad said:
Woops sorry buddy. I interchanged I2 and I3.

The correct equations for this network are:

-E1 +I1R1 +I2R2 +E2 +I1R4 = 0
-E2 -I2R2 +I3R3 = 0
I1 = I2 +I3
lol :( nope not it i have the answer, it is the 2nd equation that is giving me trouble to get. i need to understand this for coming exam. I asked my TA and he said it is not a straight forward thing
 
Last edited:
  • #16
Alt+F4 said:
lol :( nope not it i have the answer, it is the 2nd equation that is giving me trouble to get. i need to understand this for coming exam. I asked my TA and he said it is not a straight forward thing

Are you saying that what I posed (the last one) is not the right answer?

If that's the case, let me know what the right one is.


It is VERY straightforward... I have no idea why your TA is saying that. You basically have to remember two things.

1) the voltage around a loop is equal to zero.
2) current in equals current out.
 
  • #17
Alt+F4 said:
lol :( nope not it i have the answer, it is the 2nd equation that is giving me trouble to get. i need to understand this for coming exam. I asked my TA and he said it is not a straight forward thing
Never mind, for some reason it wouldn't accept my answer but i kept clickin ok and all of a sudden it took it. I had the equation from the start way before i did this thread. It wouldn't just accept my answer. Damn computers. Thanks for ur help
 
  • #18
I'm almost positive the last answer I put down is correct... if I'm wrong, I'm either having a huge space out (I did already with the switching of I2 and I3) or I have no idea what I'm doing... (which I hope is not the case ;)

oh...
so what is the answer?
 
Last edited:
  • #19
FrogPad said:
I'm almost positive the last answer I put down is correct... if I'm wrong, I'm either having a huge space out (I did already with the switching of I2 and I3) or I have no idea what I'm doing... (which I hope is not the case ;)

oh...
so what is the answer?
Answer was .031 but i had .03064435 even though it says on top to keep all sig figs and to aviod rounding
 
  • #20
ur equations were right
 
  • #21
Alt+F4 said:
ur equations were right

Good. Otherwise I'd be freaking out.


Let me know if you still need help with those circuits. Trust me. It's straightforward, I don't know why the TA is trying to make it sound like it is so difficult to you.

It's not easy until you've done hundreds of them, but even then if you are not careful you can make careless mistakes. The hard part is just about watching your signs.
 

1. What are Kirchoff's Laws?

Kirchoff's Laws are fundamental principles of electricity that govern the behavior of currents and voltages in electrical circuits. They are used to analyze and solve complex circuits.

2. What is the first law of Kirchoff's Laws?

The first law, also known as Kirchoff's Current Law (KCL), states that the algebraic sum of currents entering a node (or junction) in a circuit is equal to the algebraic sum of currents leaving that node. This law is based on the principle of conservation of charge.

3. What is the second law of Kirchoff's Laws?

The second law, also known as Kirchoff's Voltage Law (KVL), states that the algebraic sum of all voltages around a closed loop in a circuit is equal to zero. This law is based on the principle of conservation of energy.

4. What is a node in an electrical circuit?

A node is a point in an electrical circuit where two or more elements (such as resistors, capacitors, or voltage sources) are connected. It is also known as a junction or a connection point.

5. How do I apply Kirchoff's Laws to solve a circuit?

To apply Kirchoff's Laws, you need to first identify all the nodes and loops in the circuit. Then, using KCL and KVL, you can write a set of equations based on the currents and voltages at each node and loop. These equations can be solved simultaneously to find the values of unknown currents and voltages in the circuit.

Similar threads

  • Introductory Physics Homework Help
Replies
9
Views
2K
  • Introductory Physics Homework Help
Replies
18
Views
2K
  • Introductory Physics Homework Help
Replies
16
Views
924
  • Introductory Physics Homework Help
Replies
12
Views
1K
  • Introductory Physics Homework Help
Replies
4
Views
2K
  • Introductory Physics Homework Help
Replies
1
Views
1K
  • Introductory Physics Homework Help
Replies
5
Views
2K
  • Introductory Physics Homework Help
Replies
2
Views
1K
  • Introductory Physics Homework Help
Replies
6
Views
2K
  • Introductory Physics Homework Help
Replies
3
Views
1K
Back
Top